perrin.samuels
Thanks Received: 1
Forum Guests
 
Posts: 4
Joined: August 05th, 2010
 
 
 

Q11 - Unless they are used as strictly

by perrin.samuels Sat Aug 21, 2010 7:06 pm

I could use a little help with this one, please. The stimulus discusses the effects of rent-control ordinances. Correct answer choice (B) states:

"In many municipalities there is now, or eventually will be, a shortage of rental units."

The scope of this statement is not limited to municipalities governed by rent-control ordinances. How can such a general statement be inferred from the stimulus, which is limited to a discussion of rental control? Thanks!
 
giladedelman
Thanks Received: 833
LSAT Geek
 
Posts: 619
Joined: April 04th, 2010
 
This post thanked 2 times.
 
 

Re: Q11 - Unless they are used as strictly

by giladedelman Mon Aug 23, 2010 12:38 pm

Thanks for the question.

What if I said, "Many people who are vegetarians like to drink diet soda." Could I then infer that "Many people like to drink diet soda"? Yes, I could! The fact that I don't include the word "vegetarian" doesn't mean that all of a sudden I'm making a sweeping statement that includes non-vegetarians too.

That's sort of what you're missing with this problem. We're told that in many municipalities -- "specifically in all those where tenants of rent-control units have a secure hold on political power" -- those tenants are guided invariably by the desire for short-term gain, which we're told elsewhere means lower rents in exchange for the long-term disadvantage of fewer vacancies.

So "in many municipalities", the people in control of whether or not to enact rent-control ordinances are invariably going to choose to enact them, because they're interested in the short-term benefits they bring. From this, we can infer that they will incur the long-term disadvantage of rental shortages.

Again, the stimulus tells us explicitly that this is the case in many municipalities. So answer (D) doesn't enlarge the scope of the given statements at all.

(A) is unsupported. All we know is that rent control limits rent increases, but not whether it makes such increases impossible.

(B) is out of scope. We don't know what happens when the shortages arise. (Though we do know that in the municipalities totally controlled by the rent-control people, the ordinances won't be repealed.)

(C) is contradicted by the stimulus, which tells us there are several negative effects, of which this is one.

(E) is way out of scope. We have no idea about the relationship between rental unit shortage and long-term rent increases.

Does that answer your question? Please let me know if you're still puzzled about answer (D).
 
perrin.samuels
Thanks Received: 1
Forum Guests
 
Posts: 4
Joined: August 05th, 2010
 
 
 

Re: PT14, S2, Q11: Rent-control ordinances

by perrin.samuels Mon Aug 23, 2010 5:26 pm

I got it now! Thanks a lot.
 
cyruswhittaker
Thanks Received: 107
Forum Guests
 
Posts: 246
Joined: August 11th, 2010
 
 
trophy
Most Thanked
trophy
First Responder
 

Re: PT14, S2, Q11: Rent-control ordinances

by cyruswhittaker Tue Sep 07, 2010 9:50 pm

Just a fine point, but this was also an immediate hesitancy of mine regarding this problem (I felt D was too general).

Are we considering "many" essentially equivalent to "some?"

For example, if the example with vegetarians was changed to "most," it wouldn't hold anymore, right?

If most of a group (greater than half, less than all) have a certain characteristic, it would be too big of a jump to say most of a more general group has the characteristic (b/c we don't know group size) but it would be OK to say "many" or "some" have that characteristic?

Thanks!
 
giladedelman
Thanks Received: 833
LSAT Geek
 
Posts: 619
Joined: April 04th, 2010
 
This post thanked 1 time.
 
 

Re: PT14, S2, Q11: Rent-control ordinances

by giladedelman Wed Sep 15, 2010 9:44 pm

Yes, exactly. For example, if we say that most Dominicans speak Spanish, it does not follow that most Spanish-speakers are Dominican. The only thing that does follow is that some Spanish-speakers are Dominican.

And, yes, we have to treat "many" exactly as we treat "some."
 
anjelica.grace
Thanks Received: 5
Jackie Chiles
Jackie Chiles
 
Posts: 41
Joined: November 17th, 2011
 
 
 

Re: Q11 - Unless they are used as strictly

by anjelica.grace Thu May 17, 2012 1:15 pm

I'm still not convinced as to why (B) is wrong.

If tenants are motivated by short-term gain AND also are able to exercise political power to enact or repeal rent-control laws, doesn't it make sense that they'll have the controls repealed right when the long-term disadvantage (shortage of rental units) sets in?

The same power that can trigger the short-term gain can be used to avoid the long-term loss since the stimulus explicitly says "tenants ... can get rent-control ordinances enacted OR REPEALED."

That's exactly why I chose (B) over (D).

Help! This one's really frustrating me because if I accept the reasoning used to reach (D), that same reasoning would lead me to (B) since it's based on the ability to enact/repeal laws as well as motivation for short-term gain (and they clearly wouldn't want the shortage that comes later).

Basically, why on earth would they not stop the laws if they have the power to repeal them once they milked it for the short-term gains?
 
woodyguthrie
Thanks Received: 1
Forum Guests
 
Posts: 3
Joined: July 29th, 2012
 
 
 

Re: Q11 - Unless they are used as strictly

by woodyguthrie Sun Jul 29, 2012 11:31 pm

I initially got the problem wrong but when reviewing it found the right answer. In the stimulus, it states that "[rent] controls will bring about a shortage of rental units." The emphasis is mine. This makes it clear that D is right as it simply restates that fact.
 
taaron
Thanks Received: 1
Forum Guests
 
Posts: 9
Joined: October 29th, 2012
 
 
 

Re: Q11 - Unless they are used as strictly

by taaron Mon Oct 29, 2012 2:45 pm

Im with anjelica.grace on this one - if you can repeal the law before the longterm disadvantages set in, then it doesn't necessarily lead to the shortage of units (hence B and not D). if some people can't, then sure...but nowhere does it say that many people can't, as D implies...rather many people possess the power to repeal the ordinances!

Help please?

anjelica.grace Wrote:I'm still not convinced as to why (B) is wrong.

If tenants are motivated by short-term gain AND also are able to exercise political power to enact or repeal rent-control laws, doesn't it make sense that they'll have the controls repealed right when the long-term disadvantage (shortage of rental units) sets in?

The same power that can trigger the short-term gain can be used to avoid the long-term loss since the stimulus explicitly says "tenants ... can get rent-control ordinances enacted OR REPEALED."

That's exactly why I chose (B) over (D).

Help! This one's really frustrating me because if I accept the reasoning used to reach (D), that same reasoning would lead me to (B) since it's based on the ability to enact/repeal laws as well as motivation for short-term gain (and they clearly wouldn't want the shortage that comes later).

Basically, why on earth would they not stop the laws if they have the power to repeal them once they milked it for the short-term gains?
anjelica.grace Wrote:I'm still not convinced as to why (B) is wrong.

If tenants are motivated by short-term gain AND also are able to exercise political power to enact or repeal rent-control laws, doesn't it make sense that they'll have the controls repealed right when the long-term disadvantage (shortage of rental units) sets in?

The same power that can trigger the short-term gain can be used to avoid the long-term loss since the stimulus explicitly says "tenants ... can get rent-control ordinances enacted OR REPEALED."

That's exactly why I chose (B) over (D).

Help! This one's really frustrating me because if I accept the reasoning used to reach (D), that same reasoning would lead me to (B) since it's based on the ability to enact/repeal laws as well as motivation for short-term gain (and they clearly wouldn't want the shortage that comes later).

Basically, why on earth would they not stop the laws if they have the power to repeal them once they milked it for the short-term gains?
User avatar
 
tommywallach
Thanks Received: 468
Atticus Finch
Atticus Finch
 
Posts: 1041
Joined: August 11th, 2009
 
This post thanked 2 times.
 
 

Re: Q11 - Unless they are used as strictly

by tommywallach Wed Oct 31, 2012 3:57 pm

Hey Guys,

I love the life that this one has had! Seems like it's time for one of us to get involved again!

So, first let me state the problem people are seeing. The passage says that in some municipalities, people have the power to repeal rent-control laws. We also know that eventually, these laws create a situation where there's a scarcity of apartments, which might cause price increases for non rent-controlled apartments that hurt renters overall.

However, there's no reason to believe that people IMMEDIATELY repeal the laws once this long-term pain sets in. First of all, there's no way to do know if they EVER repeal the laws (the argument only says they have the power to, not that they actually do it), but beyond that, there's no way to know if they do it "as soon as shortages of rental units arise."

Remember, on any inference question (which this is), you need to be very nervous about picking anything with extreme language. "as soon as" is too immediate and certain.

Hope that helps!

-t
Tommy Wallach
Manhattan LSAT Instructor
twallach@manhattanprep.com
Image
 
zainrizvi
Thanks Received: 16
Atticus Finch
Atticus Finch
 
Posts: 171
Joined: July 19th, 2011
 
 
trophy
First Responder
 

Re: Q11 - Unless they are used as strictly

by zainrizvi Wed Mar 27, 2013 4:54 pm

For some reason I'm not completely comfortable with answer choice D.

There is now or eventually will be a shortage of rental units. But what if the renters repealed the law? I know it's a pretty big what if but it's stated in the passage that it is a possibility. Without knowing whether they repeal it or not, how can we infer answer choice D? Or is it that in these kinds of situations we just see which answer choice is making the least assumptions
User avatar
 
tommywallach
Thanks Received: 468
Atticus Finch
Atticus Finch
 
Posts: 1041
Joined: August 11th, 2009
 
 
 

Re: Q11 - Unless they are used as strictly

by tommywallach Wed Mar 27, 2013 11:05 pm

Hey Zain,

Two things:

1) You are not being asked to pick a perfect answer. You're being asked to pick the best answer. Sometimes that answer will be kind of terrible. You are absolutely right. If the laws were repealed, this cycle could end, and then (D) would not occur. But...

2) ...that's actually a pretty big if. All we know for certain right now is that "in many municipalities" there is a problem which, if left unchecked, leads to a given problem. So it's fair to assume that problem will come.

So, in closing, whenever you find yourself not loving the right answer, always start by asking yourself if there's really a better one. If there isn't, then it really doesn't matter how bad/full of holes the correct answer is. It's what you've got to work with!

-t
Tommy Wallach
Manhattan LSAT Instructor
twallach@manhattanprep.com
Image
 
shodges
Thanks Received: 0
Jackie Chiles
Jackie Chiles
 
Posts: 41
Joined: August 23rd, 2011
 
 
 

Re: Q11 - Unless they are used as strictly

by shodges Mon Apr 15, 2013 9:00 am

zainrizvi Wrote:For some reason I'm not completely comfortable with answer choice D.

There is now or eventually will be a shortage of rental units. But what if the renters repealed the law? I know it's a pretty big what if but it's stated in the passage that it is a possibility. Without knowing whether they repeal it or not, how can we infer answer choice D? Or is it that in these kinds of situations we just see which answer choice is making the least assumptions


Also, I feel that I should point out that we only want what MUST be true from the stimulus. We KNOW that there will be a shortage of rental units, so we can stop there and not worry about whether that will ultimately change.
 
JBLSAT423
Thanks Received: 0
Vinny Gambini
Vinny Gambini
 
Posts: 2
Joined: October 27th, 2014
 
 
 

Re: Q11 - Unless they are used as strictly

by JBLSAT423 Mon Oct 27, 2014 8:10 pm

Also, I feel that I should point out that we only want what MUST be true from the stimulus. We KNOW that there will be a shortage of rental units, so we can stop there and not worry about whether that will ultimately change.


Actually I feel like we don't know that there will be a shortage of rental units. The stimulus says "Unless they are used as strictly temporary measures". What if ever municipality or region uses strictly temporary measures? I mean, you would think that some municipalities would incur shortages of rental units, but can we infer that from what we have in the passage?

I feel like we can't. Like if one were to say, "Oh well that is a stretch to think that no municipality will have shortages", then I would say, "Well, I think it is a stretch to think that no ordinances are repealed as soon as shortages arise (or answer choice B)".

Anyway, I just feel like there have been better questions than this, I mean it is from PT 14.

Or maybe I'm just bitter because I went -9 on on this LR section.
User avatar
 
tommywallach
Thanks Received: 468
Atticus Finch
Atticus Finch
 
Posts: 1041
Joined: August 11th, 2009
 
 
 

Re: Q11 - Unless they are used as strictly

by tommywallach Thu Oct 30, 2014 11:06 pm

Hey JBL,

Don't be bitter! We all have off days. And I agree that old tests are janky.

-t
Tommy Wallach
Manhattan LSAT Instructor
twallach@manhattanprep.com
Image
User avatar
 
Mab6q
Thanks Received: 31
Atticus Finch
Atticus Finch
 
Posts: 290
Joined: June 30th, 2013
 
 
 

Re: Q11 - Unless they are used as strictly

by Mab6q Mon Nov 17, 2014 10:10 pm

Just as we don't know whether the ordinances will be replaced as soon as shortages arise, we don't know whether there will be a shortage of rental units because as the argument allows, the rent ordinances could be strictly temporary measures. I chose D, but I really can't justify it over B.
"Just keep swimming"
 
christine.defenbaugh
Thanks Received: 585
Atticus Finch
Atticus Finch
 
Posts: 536
Joined: May 17th, 2013
 
This post thanked 1 time.
 
 

Re: Q11 - Unless they are used as strictly

by christine.defenbaugh Mon Nov 24, 2014 8:17 pm

Thanks for posting, Mab6q!

I understand your skepticism here, completely. However, we can actually prove (or come darn close to proving) that there will be a shortage of rental units in at least one municipality.

Let's start with what we know for certain, then layer in the conditionals as they apply.

    FACT: There are many municipalities that have rent-control tenants in power.
    FACT: In those particular municipalities, short-term gain determines what they enact/repeal.
    FACT: Rent-control has a short-term gain - smaller rent increases.

From these facts, I can conclude that these municipalities we're talking about currently have rent-control, and the rent-control tenants in power will continue to vote to keep the rent control (since it has short term gain). Now, that means the rent-control is NOT a "strictly temporary measure"!

    CONDITIONAL: If not strictly temporary, rent control has negative effects that include shortage.

Since these municipalities must have rent-control that is not strictly temporary, they must eventually hit that long run negative effect of a rental shortage!

Notice that there's no real chance, in these situations, that these rent-control tenants in power will ever vote to repeal, because we're told that short-term gain is always what they'll vote for. Even if we actually hit the rental shortage, fixing it would probably be a long-term gain, not a short term gain. There will still ALWAYS be a short term gain in the smaller rent increases, so that is always what they'll choose.

Surprisingly tough question for a #11. I'll blame the early preptest for that oddness. It's actually not a bad question, but it's harder than I'd expect for where it is in the section.

Does this help clear up a few things?
User avatar
 
Mab6q
Thanks Received: 31
Atticus Finch
Atticus Finch
 
Posts: 290
Joined: June 30th, 2013
 
 
 

Re: Q11 - Unless they are used as strictly

by Mab6q Wed Dec 31, 2014 1:36 am

Wow Christine, that made perfect sense. Thank you so much!
"Just keep swimming"
 
pbookworm
Thanks Received: 0
Vinny Gambini
Vinny Gambini
 
Posts: 7
Joined: March 19th, 2015
 
 
 

Re: Q11 - Unless they are used as strictly

by pbookworm Thu Apr 09, 2015 8:00 pm

I'm still a little confused on this question - if anyone can help. I picked D since it was the best out of the 5, but is it provable?

We are told that in a certain # of municipalities, it is always the desire for short term gain (smaller rent increases) that leads tenants to exercise power. I know that the short term gain (smaller rent increases) leads to a disadvantage (shortage of rental units).

But we don't know that the tenants exercise that power. (Or do we?) Thus, if we don't know that they exercise that power (and thus we don't know if they have smaller rent increases), how do we know that there is a shortage of rental units?!

Thanks in advance!